boards 5

Lakukan tugas rumah & ujian kamu dengan baik sekarang menggunakan Quizwiz!

After ordering three albuterol treatments for a status asthmaticus patient, the doctor then orders continuous nebulization of 25 mg of albuterol (1:200) for an asthmatic patient. How many mL of albuterol would you place in the nebulizer chamber? Select one: A. 5.0 mL B. 0.25 mL C. 2.5 mL D. 0.5 mL

10 mg/1 mL is a one percent solution. Therefore, a 1:200 or 0.5% strength given in the question is a 5 mg / 1 mL solution. To solve this, set up the following simple equation and solve for X: 5 mg / 1 mL = 25 mg / X mL X = 25/5 = 5 mL The correct answer is: 5.0 mL

Which of the following arrhythmias are considered potentially lethal? Ventricular fibrillation Ventricular tachycardia 3rd degree heart block Supraventricular tachycardia A. Yes Yes Yes No B. Yes Yes Yes Yes C. Yes No Yes No D. No Yes No Yes

3rd degree heart block (atrial-ventricular conduction disassociation), ventricular tachycardia, and ventricular fibrillation are all considered potentially lethal arrhythmias because they all can cause either severe reduction or complete cessation of cardiac output, In supraventricular tachycardia (SVT) the ventricles normally are protected against excessively high rates by the atrioventricular node, which allows only a proportion of the rapid impulses to pass through. For this reason, SVT is not considered a life-threatening or lethal arrhythmia. The correct answer is: A

The data below were obtained while an adult patient was receiving controlled mechanical ventilation with an FIO2 of 0.50 (PIP = peak pressure; BP = blood pressure) Time PIP cm H2O PEEP cm H2O PaO2 torr PvO2 torr BP mm Hg 5:00 40 0 43 28 140/90 5:20 45 6 53 39 130/90 5:40 46 8 58 43 120/90 6:00 50 10 65 45 120/80 6:20 52 12 75 42 105/70 Based on these date, which PEEP level would you recommend for this patient? Select one: A. 6 cm H2O B. 8 cm H2O C. 10 cm H2O D. 12 cm H2O

A PEEP study is done to determine the 'best PEEP.' One approach (as here) is to identify the PEEP pressure which results in the best tissue oxygenation with the least potential for barotrauma. In this example, the best tissue oxygenation occurs at the highest PvO2 (10 cm H2O PEEP), without any hemodynamic compromise (normal BP). The correct answer is: 10 cm H2O

A patient with asthma reports experiencing symptoms > 2 days/week; is awakened at night with wheezing about once a week, and uses her rescue inhaler about three times a week. Her peak flow is 70% of predicted and she has been prescribed oral steroids three times over the past nine months. How well controlled is her asthma? Select one: A. well controlled B. not well controlled C. very poorly controlled D. uncontrolled

A patient's asthma is considered not well controlled if symptoms occur > 2 days/week; nighttime awakenings occur 1-3x/week; there is some limitation to normal activities; short-acting beta-agonists are used > 2 days/week; the FEV1 or peak flow is 60 - 80% predicted/personal best; and exacerbations (requiring oral steroids) occur ≥ 2x/year. The correct answer is: not well controlled

A patient with asthma reports experiencing symptoms ≤ 2 days/week; is awakened at night with wheezing about once a month, and uses her rescue inhaler about once a week. Her FEV1 is 85% of predicted. How well controlled is her asthma? Select one: A. well controlled B. not well controlled C. very poorly controlled D. uncontrolled

A patient's asthma is considered well-controlled if symptoms occur ≤ 2 days/week; nighttime awakening ≤ 2x/month; there is no interference with normal activities; short-acting beta-agonists are used ≤ 2 days/week; the FEV1 or peak flow is > 80% predicted/personal best; and exacerbations (requiring oral steroids) occur once a year or less. The correct answer is: well controlled

A patient with thick secretions who is receiving an aerosol therapy with 10% acetylcysteine (Mucomyst) reports shortness of breath after her treatments. Which of the following would you recommend for this patient? Select one: A. dilute the acetylcysteine to 5% mixture B. add albuterol (Proventil) to the aerosol mixture C. add cromolyn sodium (Intal) to the aerosol mixture D. discontinue the acetylcysteine treatments

Acetylcysteine (Mucomyst) can be very irritating to the airways and can cause bronchospasm. For this reason, it normally is given with or after a bronchodilator. The correct answer is: add albuterol (Proventil) to the aerosol mixture

Which of the following statement about chest compressions during neonatal resuscitation is correct? Select one: A. compressions and ventilations should be delivered simultaneously B. the compression to ventilation ratio should be 30:2 C. compressions should exceed 1/2 the AP chest diameter D. there should be at least 90 compressions per minute

According to the American Heart Association (AHA) guidelines for compression and ventilation of neonates, there should be a 3:1 ratio of compressions to ventilations with 90 compressions and 30 breaths to achieve approximately 120 events per minute to maximize ventilation at an achievable rate. Each event is thus allotted about 1⁄2 second (requiring a compression rate > 100/min between breaths), with exhalation occurring during the first compression after each ventilation. The correct answer is: there should be at least 90 compressions per minute

During combined ventilation+chest compressions of newborn infants requiring resuscitation, what is the proper compression to ventilation ratio? Select one: A. 3:1 B. 5:1 C. 1:3 D. 1:5

According to the American Heart Association (AHA) guidelines for neonatal/infant resuscitation, there should be a 3:1 ratio of compressions to ventilations with 90 compressions and 30 breaths to achieve approximately 120 events per minute to maximize ventilation at an achievable rate. Each event is thus allotted about 1⁄2 second (requiring a compression rate > 100/min between breaths), with exhalation occurring during the first compression after each ventilation. The correct answer is: 3:1

Which of the following drugs are used to treat patients with central sleep apnea syndrome? Acetazolamide (Diamox) Propofol (Diprivan) Theophylline (Theo-Dur) A. Yes Yes No B. Yes No Yes C. No Yes Yes D. Yes Yes Yes

Acetazolamide (Diamox; a carbonic anhydrase inhibitor) and theophylline Theo-Dur; a phosphodiesterase inhibitor) both have respiratory stimulant properties and are one treatment option for patients with central sleep apnea, especially that associated with Cheyne-Stokes breathing and heart failure. Sedative-hypnotics such as zolpidem (Ambien) and triazolam (Halcion) are also an option, but only for patients not at risk for respiratory depression (close clinical follow-up is required for patients using these agents). Propofol (Diprivan) is a short-acting hypnotic agent used to induce general anesthesia and provide procedural sedation. It is not used to treat central sleep apnea. The correct answer is: B

A patient is receiving postural drainage and percussion in the Trendelenburg position. Therapy should be terminated if which of the following develops? Select one: A. hemoptysis B. productive coughing C. mild dyspnea D. flushing

Active hemoptysis is an absolute contraindication for chest percussion and postural drainage and thus grounds for immediately terminating the therapy. The correct answer is: hemoptysis

You find an adult patient unresponsive and not breathing. After confirming that no carotid pulse is present and calling for a defibrillator, you should: Select one: A. establish an open airway B. begin chest compressions C. attach an ECG monitor D. give two rescue breaths

After verifying unresponsiveness and asking for a defibrillator, your first action should be to provide 30 chest compressions. The current AHA recommendation is to begin chest compressions before airway access or rescue breathing, i.e., C-A-B. Delays in or and interruptions of chest compressions should be minimized. Moreover, chest compressions can be started almost immediately, while establishing an airway and providing ventilation takes time. Beginning CPR with 30 compressions rather than 2 ventilations leads to a shorter delay to first compression. The correct answer is: begin chest compressions

A patient with emphysema receiving volume control A/C ventilation develops signs of air trapping (auto-PEEP). The doctor does not want to switch to SIMV. Which of the following actions would help resolve this problem? Select one: A. decrease the length of time available for exhalation B. increase the preset frequency of breathing by 30% C. increase the length of time available for exhalation D. increase the inspiratory time by decreasing the flow

Air trapping/auto-PEEP during positive pressure ventilation is most likely when insufficient time is provided for exhalation. Although I:E ratios of 1:2 or 1:3 are generally sufficient to ensure complete exhalation of patients with normal lungs, lower ratios (e.g., 1:4, 1:5) may be required in some patients with severe COPD. Alternatively, small amounts of machine PEEP (50-80% of the measured auto-PEEP level) can be used to help overcome auto-PEEP. The correct answer is: increase the length of time available for exhalation

A 45-year-old patient with asthma is prescribed 0.3 mL of albuterol (Proventil) in 3 mL normal saline via small volume nebulizer. Before initiating therapy, you note from chart review that the patient is severely hypertensive and has been experiencing episodes of superventricular tachycardia. You should do which of the following? Select one: A. administer the treatment as ordered B. postpone the treatment and consult the physician C. dilute the albuterol with extra normal saline D. decrease the amount of albuterol administered

Albuterol is a beta-adrenergic drug that can increase heart rate and blood pressure. For this reason, hypertension and tachycardia are contraindications to its administration. Whenever a contraindication exists to drug administration, you normally should postpone the treatment and consult with the ordering physician. The correct answer is: postpone the treatment and consult the physician

A 10-year-old female patient with cystic fibrosis is admitted to the hospital. She has very thick yellow-green secretions, coarse rhonchi and wheezing in both lower lobes. The doctor asks you to recommend an aerosolized drug regimen to help with her breathing problems. Which of the following would you recommend? Albuterol (Proventil) Tobramycin (TOBI) Dornase alpha (Pulmozyme) Hypertonic saline A. Yes Yes Yes No B. Yes Yes Yes Yes C. Yes No Yes No D. No Yes No Yes

Albuterol should be administered first to treat probable bronchospasm (as indicated by wheezing) and open the airways for administration of the other drugs. Dornase alpha (Pulmozyme) helps break down DNA-rich secretions (typical of CF) and should be administered after albuterol. Hypertonic saline draws water onto the respiratory tract mucosa and can facilitate coughing and airway clearance (and also used in CF patients who cannot tolerate Pulmozyme). Last, tobramycin (TOBI) should be administered to help combat Pseudomonas infection. Some form of airway clearance therapy may also be needed to help the patient mobilize and remove her secretions. The correct answer is: B

All patients with asthma should be prescribed a: Select one: A. leukotriene modifier B. short-acting beta-agonist C. oral corticosteroid D. long-acting beta-agonist

All patients with asthma should be prescribed a short-acting beta-agonist inhaled bronchodilator as needed (PRN). Assuming good asthma education, elimination of triggers and management of comorbidities, drug management is then tailored to the severity of disease and its degree of control. The correct answer is: short-acting beta-agonist

As part of a weaning protocol, you switch a 85 kg (187 lb) patient from SIMV at 4/min to 5 cm H2O CPAP and an FIO2 of 0.45. Twenty minutes after switching to CPAP, you obtain the following data: pH = 7.24 Spon Resp Rate 36/min PaCO2 = 53 torr MIP/NIF -23 cm H2O PaO2 = 49 torr Tidal Volume 275 mL HCO3 = 22 mEq/L Vital Capacity 1.2 L Which of the following actions would you recommend at this time? Select one: A. administer sodium bicarbonate B. continue CPAP, but raise the FIO2 to 0.55 C. increase the CPAP to 8 cm H2O D. put the patient back on SIMV

Although the MIP and VC are adequate, the high PCO2, high respiratory rate with low tidal volume (RSBI = 36/.275 = 131) and dangerously low pH and PaO2 indicate a failed weaning attempt. You should put the patient back on ventilatory support and try again when appropriate. The correct answer is: put the patient back on SIMV

A physician requests your advice in managing a 49-year-old obese male patient with confirmed obstructive sleep apnea. Which of the following would you recommend? Select one: A. placement of a tracheal button B. use of respiratory stimulants C. administration of CPAP at night D. surgical uvulopalatopharyngoplasty

Among all sleep apnea treatment modalities, CPAP is generally the most successful, least hazardous, and best tolerated. CPAP, usually administered via a nasal device, distends the oropharynx, thereby preventing occlusion by the tongue and soft palate. The correct answer is: administration of CPAP at night

In individuals with disorders characterized by an increase in airway resistance, such as emphysema, which of the following breathing patterns results in the minimum work? Select one: A. deep breathing B. slow breathing C. shallow breathing D. rapid breathing

An increase in airway resistance increases the frictional work of breathing, i.e. the pressure difference due to air flow. Decreasing the rate of breathing will decrease the pressure difference due to air flow. Thus in these patients a slow breathing pattern (decreased flows) will result in the minimum work. The correct answer is: slow breathing

A post cardiac surgery patient receiving mechanical ventilation uses a writing slate to complain to you of pain in the area of her incision and where her chest tubes were inserted. Which of the following drugs would you recommend for this patient? Select one: A. haloperidol (Haldol) B. vecuronium (Norcuron) C. midazolam (Versed) D. fentanyl (Sublimaze)

Analgesics should be prescribed for any patient receiving mechanical ventilation who may be experiencing pain. IV narcotics are usually prescribed for these patients. Morphine is the drug of choice for patients with stable cardiovascular status. If the patient' cardiovascular status is unstable, the histamine-associated hypotension that morphine may cause can be avoided by using fentanyl (Sublimaze) or hydromorphone (Dilaudid). Neither haloperidol (an antipsychotic), midazolam (a benzodiazepine sedative), nor vecuronium (a neuromuscular blocking agent) have any affect on pain. The correct answer is: fentanyl (Sublimaze)

During CPR of an adult patient in the emergency department, a gas-powered resuscitator fails to deliver adequate tidal volumes. Your immediate response would be to: Select one: A. inspect the unit B. call for help C. use a mechanical ventilator D. change to a bag-valve-mask

Any interruption of ventilation or cardiac compressions during CPR can negatively affect the outcome. If a gas-powered resuscitator fails to deliver adequate tidal volumes during CPR, you should immediately request a bag-valve-mask (manual resuscitator) and continue ventilation with 100% oxygen. The correct answer is: change to a bag-valve-mask

When assessing a patient after a treatment, you note a significant deterioration in vital signs. The most appropriate action in this case is to: Select one: A. call for the institution's rapid response team B. report the findings to the next shift of respiratory therapy staff C. chart the findings as an unexpected response to therapy D. orally communicate the findings to the patient's physician

Any unexpected response to therapy or adverse effects noted when charting a patient encounter should also be communicated orally to the patient's physician and nurse. In general, the more serious the problem, the sooner these key people should be informed. However, if it is clear that the patient's vital signs are deteriorating, do not wait to inform the doctor. Instead, call for your institution's rapid response team. The correct answer is: call for the institution's rapid response team

A physician orders 2 L/min O2 via simple mask to a 65 year-old patient with chronic hypercapnia. The correct action at this time would be to: Select one: A. recommend a flow of at least 5 L/min to wash out CO2 B. recommend that the mask be changed to a cannula at 2 L/min C. not apply the oxygen until contacting the medical director D. not carry out the order until the attending nurse agrees

As a guide for O2 therapy in patients with chronic hypercapnia, the objective should be to maintain a PaO2 between 50-60 mm Hg. Generally, this will maintain adequate tissue oxygenation, while minimizing the likelihood of inducing hypoventilation. This is best achieved via either low-flow nasal O2 (1-3 L/min) or a low concentration (24-28%) air entrainment mask. The correct answer is: recommend that the mask be changed to a cannula at 2 L/min

A premature 1850 g newborn infant is receiving 40% oxygen in an oxyhood. A chest X-ray shows a ground-glass appearance bilaterally, with air bronchograms. An arterial blood gas reveals the following: pH = 7.36 PaCO2 = 44 torr PaO2 = 45 torr HCO3 = 24 mEq/L BE = 0 mEq/L What would you recommend at this time? Select one: A. intubate and initiate 10 cm H2O CPAP B. intubate and initiate mechanical ventilation C. administer 5 cm H2O nasal CPAP with an FIO2 of 0.50 D. place the neonate in an Isolette with an FIO2 of 0.60

As suggested by the X-ray and blood gas, the likely problem is Respiratory Distress Syndrome. Since the blood gas indicates adequate ventilation but poor oxygenation (P/F ratio of about 100), a trial of noninvasive CPAP is the place to begin. 5 cm H2O is a reasonable starting point, with the FIO2 slightly elevated (0.50) to ensure adequate oxygenation. Intubation is not yet indicated, and raising the FIO2 alone will not help overcome the shunting that occurs in IRDS. The correct answer is: administer 5 cm H2O nasal CPAP with an FIO2 of 0.50

A 9 year old patient with asthma has symptoms 3-4 times/week, with night-time problems occurring a couple times a month. His FEV1/FVC is 83% predicted. In addition to a short-acting beta agonist PRN for symptom relief, which of the following drugs would you recommend to help control his condition over the long-term? Select one: A. Cromolyn sodium (Intal) 2 puffs 4x daily B. Salmeterol (Serevent) 2 puffs 2x daily C. Beclomethasone (Vanceril) 2 puffs 2x daily D. Formoterol (Foradil) 1 cap (DPI) 2x daily

Based on the symptoms and PFT results, the patient has mild persistent asthma. Long-term control of mild persistent asthma is best achieved using a single anti-inflammatory agent, either a low dose corticosteroid like beclomethasone or a mast cell inhibitor like cromolyn sodium (Intal). To avoid the potential side effects of steroids, children usually begin with a trial of cromolyn. Long-acting beta agonists (LABAs) like formoterol and salmeterol are prescribed for moderate to severe persistent asthma. Used alone, LABAs may increase the risk of death in patient with asthma. For this reason, LABAs should only be used in conjunction with an inhaled corticosteroid for patients with asthma. The correct answer is: Cromolyn sodium (Intal) 2 puffs 4x daily

A surgeon orders an increase in PEEP from 6 to 10 cm H2O for a post-op patient receiving mechanical ventilation. After raising the PEEP, you note a rapid fall in the patient's blood pressure and a rapid rise in her heart rate. Which of the following actions would you recommend to the surgeon? Select one: A. increase the FIO2 by 10% B. administer a vasopressor C. return the PEEP to 6 cm H2O D. obtain a stat arterial blood gas

Because PEEP increases intrathoracic pressure, it can impede venous return and decrease cardiac output. The drop in this patient's blood pressure and rise in her heart rate indicates that the increase in PEEP is decreasing her cardiac output. In general, whenever an intervention has a bad effect, the first action should be to reverse the intervention, in this case by returning the PEEP to 6 cm H2O. The correct answer is: return the PEEP to 6 cm H2O

A nurse complains that her trach patient's secretions are thick and difficult to suction. You note that the patient is receiving 40% oxygen via a jet nebulizer and tracheotomy collar and that the secretions appear mucoid in nature. You also note a lack of skin turgor and dry/cracked lips. What of the following would you recommend in this case? Select one: A. instilling 3-5 mL of saline before suctioning B. increasing the vacuum pressure to 180 mm Hg C. providing adequate systemic hydration D. administering a broad spectrum antibiotic

Bedside assessment of this patient indicates inadequate hydration (loss of skin turgor and dry/cracked lips). The best way to overcome problems with thick secretions is to provide adequate systemic hydration. Instilling saline prior to suctioning is controversial and certainly does not treat the problem of inadequate hydration (nor does simply increasing the suction pressure). Also given the description of the secretions, antibiotic adminstration probably is not warranted. The correct answer is: providing adequate systemic hydration

After weaning and extubation, a patient on a cool aerosol mask at 40% O2 develops moderate hypoxemia and hypercapnia, with a falling pH. Which of the following actions would you recommend at this time? Select one: A. increase the nebulizer oxygen concentration to 60% B. apply bi-level positive airway pressure via mask C. administer 50 mEq IV sodium bicarbonate D. re-intubate and apply volume control ventilation

Bi-level positive airway pressure (BiPAP) is a good option to help avoid reintubation of patients who develop mild to moderate hypercapnia or hypoxemia after extubation. It also is used to (1) avoid intubation of COPD patients requiring ventilatory support for acute on chronic ventilatory failure, (2) treat CHF/pulmonary edema (short term application), and (3) manage obstructive and central sleep apnea (nocturnal application). The correct answer is: apply bi-level positive airway pressure via mask

A 75 kg (165 lb) patient is volume control SIMV with a mandatory rate of 8/min, an FIO2 of 0.40, and a tidal volume of 600 mL. Arterial blood gas results are as follows: pH = 7.28 PaCO2 = 57 torr PaO2 = 80 torr HCO3 = 26 mEq/L BE = -1 mEq/L Based on the patient's clinical condition, which of the following actions should be taken? Select one: A. initiate 5 cm H2O PEEP B. increase the FIO2 to 0.50 C. increase the SIMV rate to 12/mm D. increase the tidal volume to 900 mL

Blood gas analysis indicates adequate oxygenation, but acute respiratory acidosis. To restore the pH back toward normal, you must increase the minute ventilation. To increase the minute ventilation, you could either increase the SIMV rate or increase the set tidal volume. However, the VT already is about right for this patient (8 mL/kg), so the best option is to increase the mandatory SIMV rate. During SIMV, you usually change the minute ventilation by changing the rate. The correct answer is: increase the SIMV rate to 12/mm

A patient is receiving aerosol treatments with albuterol (Proventil) q4h following an uneventful cholecystectomy. Patient assessment reveals clear breath sounds bilaterally. The patient has no history of smoking and takes deep breaths with strong coughs voluntarily throughout the day. What should you recommend? Select one: A. discontinuing the aerosol treatment B. continuing the treatment as ordered C. postural drainage and percussion D. a MDI with albuterol

Bronchodilator treatments are used to reverse bronchospasm. The patient in this case has normal airways and does not need a bronchodilator. It should be discontinued. Moreover, the strong cough indicates that the patient has good airway clearance and probably only needs encouragement to continue directed coughing, deep breathing and ambulation to prevent postop problems. The correct answer is: discontinuing the aerosol treatment

You would recommend continuous positive airway pressure (CPAP) to manage which one of the following problems? Select one: A. obstructive sleep apnea B. drug overdose C. hypercapnic respiratory failure D. central sleep apnea

CPAP involves the application of positive pressure to the airway throughout the spontaneous breathing cycle. It is indicated for patients: (1) with CHF/cardiogenic pulmonary edema (short term application); (2) with obstructive sleep apnea (nocturnal application); and (3) with adequate spontaneous ventilation but severe hypoxemia due to shunting (e.g., ARDS, IRDS). CPAP is not indicated for patients with inadequate ventilation/CO2 removal, such as those with central sleep apnea, drug overdose, and hypercapnic respiratory failure. The correct answer is: obstructive sleep apnea

A COPD patient is receiving sustained-release theophylline treatments. Which of the following is a possible adverse effect of this therapy? Select one: A. hyperkalemia B. hypoglycemia C. cardiac arrhythmias D. respiratory alkalosis

Cardiovascular adverse effects of theophylline include hypotension and arrhythmias (e.g., sinus tachycardia, PVCs, atrial fibrillation, atrial flutter, supraventricular tachycardia, V-tach and V-fib). Gastrointestinal effects include nausea, vomiting, diarrhea, and abdominal pain or cramping. Metabolic effects include hypokalemia, hyperglycemia, hypercalcemia, rhabdomyolysis, and acidosis. Neurologic effects include headaches, restlessness, tremors, disorientation, hallucinations, insomnia, and seizures. To avoid adverse effects, the serum levels of theophylline should be titrated to maintain the therapeutic range of 10-20 mg/L. The correct answer is: cardiac arrhythmias

Which of the following can cause pulseless electrical activity (PEA)? Select one: A. hyperthermia B. altered mental status C. hypervolemia D. cardiac tamponade

Causes of pulseless electrical activity (PEA) include massive pulmonary embolism, cardiac tamponade, tension pneumothorax, HYPOvolemia, hypoxia, hypothermia, drug overdose, hyperkalemia, pre-existing acidosis and acute, massive MI. The correct answer is: cardiac tamponade

Which of the following medication you would recommend to help quiet an ICU patient who is breathing asynchronously on a ventilator? Select one: A. fluoxetine (Prozac) B. cisatracurium (Nimbex) C. dextroamphetamine (Dexedrine) D. fentanyl (Sublimaze)

Common medications used to sedate mechanically ventilated patients include benzodiazepines like midazolam (Versed); hypnotics like propofol (Diprivan); and alpha-2 agonists like dexmetatomidine (Precedex) or clonidine (Catapres). Opioid analgesics like fentanyl (Sublimaze) or remifentanil (Ultiva) also can be used as sedating agents. Cisatracurium (Nimbex) is a neuromuscular blocking agent (not a sedative!), dextroamphetamine (Dexedrine) is a stimulant, and fluoxetine (Prozac) an antidepressant. The correct answer is: fentanyl (Sublimaze)

An patient in ICU receiving noninvasive ventilation is agitated and having difficulty accepting the nasal interface. Which of the following medications would you recommend to sedate this patient? Select one: A. albuterol (Proventil) B. diltiazem (Cardizem) C. cisatracurium (Nimbex) D. dexmedetomidine (Precedex)

Common sedatives used to calm patient in ICU include benzodiazepines like midazolam (Versed); hypnotics like propofol (Diprivan); and alpha-2 agonists like clonidine (Catapres) or dexmetatomidine (Precedex). Cisatracurium is a neuromuscular blocking agent, not a sedative. Albuterol is a bronchodilator and diltiazem is a calcium channel blocker used in treating angina, hypertension and certain cardiac arrhythmias. The correct answer is: dexmedetomidine (Precedex)

Which of the following are clinical signs of reduced cardiac output or poor tissue perfusion? Select one: A. bradycardia B. bounding pulses C. excess urine output D. cool extremities

Clinical signs of reduced cardiac output and poor tissue perfusion include hypotension, tachycardia, cool extremities with weak or absent pulses, poor capillary refill, acrocyanosis, reduced urinary output, and--if severe--a semiconscious or comatose state, The correct answer is: cool extremities

Which of the following are clinical signs of reduced cardiac output or poor tissue perfusion? Select one: A. bradycardia B. bounding pulses C. decreasd urine output D. warm extremities

Clinical signs of reduced cardiac output and poor tissue perfusion include hypotension, tachycardia, cool extremities with weak or absent pulses, poor capillary refill, acrocyanosis, reduced urinary output, and--if severe--a semiconscious or comatose state, The correct answer is: decreasd urine output

An patient in ICU receiving noninvasive ventilation is agitated and having difficulty accepting the nasal interface. Which of the following medications would you recommend to sedate this patient? Select one: A. albuterol (Proventil) B. diltiazem (Cardizem) C. cisatracurium (Nimbex) D. midazolam (Versed)

Common sedatives used to calm patient in ICU include benzodiazepines like midazolam (Versed); hypnotics like propofol (Diprivan); and alpha-2 agonists like dexmetatomidine (Precedex) or clonidine (Catapres). Cisatracurium is a neuromuscular blocking agent, not a sedative. Albuterol is a bronchodilator and diltiazem is a calcium channel blocker used in treating angina, hypertension and certain cardiac arrhythmias. The correct answer is: midazolam (Versed)

A patient able to maintain adequate spontaneous ventilation has PaO2 of 42 mm Hg while receiving oxygen via a nonrebreathing mask set at 12 L/min. Which of the following would you recommend to the patient's physician? Select one: A. initiate continuous positive airway pressure B. increase the mask oxygen flow to 15 L/min C. administer IPPB treatments q2h with 100% O2 D. switch to a partial rebreathing mask at 15 L/min

Continuous positive airway pressure (CPAP) is the appropriate mode of support for patients with adequate spontaneous ventilation, but who continue to have severe hypoxemia due to physiologic shunting. As a rule of thumb, severe shunting exists when the PaO2 of a patient cannot be maintained above 60 mm Hg with an FIO2 greater than 0.60, as provided by a nonrebreathing mask at 12 L/min (the '60/60' rule). The correct answer is: initiate continuous positive airway pressure

Which of the following clinical findings are consistent with cor pulmonale? Select one: A. left axis deviation on ECG B. distention of the neck veins C. high pulmonary artery wedge pressue D. bilateral infiltrates on chest X-ray

Cor pulmonale is right heart failure associated with primary lung disease such as COPD. The common underlying problem is pulmonary hypertension, not due to left heart failure (thus a normal PCWP/PAWP). Clinical findings include an enlarged liver (hepatomegaly), right upper abdominal pain or tenderness, jugular venous distention/elevated CVP, and dependent edema. The ECG typically reveals right axis deviation and an increase in P wave amplitude in leads 2, 3, and aVF (the "P-pulmonale" pattern). The chest X-ray may show engorgement of the central pulmonary arteries and right ventricular enlargement. Bilateral infiltrates would be more characteristic of left not right heart failure. Other findings such as cyanosis or dyspnea may reflect the underlying lung disease. The correct answer is: distention of the neck veins

Which of the following clinical findings are consistent with core pulmonale? Select one: A. left axis deviation on ECG B. gravity-dependent edema C. high pulmonary artery wedge pressue D. bilateral infiltrates on chest X-ray

Core pulmonale is right heart failure associated with primary lung disease such as COPD. The common underlying problem is pulmonary hypertension, not due to left heart failure (thus a normal PCWP/PAWP). Clinical findings include an enlarged liver (hepatomegaly), right upper abdominal pain or tenderness, jugular venous distention/elevated CVP, and dependent edema. The ECG typically reveals right axis deviation and an increase in P wave amplitude in leads 2, 3, and aVF (the "P-pulmonale" pattern). The chest X-ray may show engorgement of the central pulmonary arteries and right ventricular enlargement. Bilateral infiltrates would be more characteristic of left not right heart failure. Other findings such as cyanosis or dyspnea may reflect the underlying lung disease. The correct answer is: gravity-dependent edema

Which of the following aerosolized agents would you recommend to facilitate airway clearance in a patient with cystic fibrosis (CF) who cannot tolerate DNase? Select one: A. hypertonic saline B. albuterol (Proventil) C. dexamethasone (Decadron) D. tobramycin (TOBI)

DNase is a mucokinetic agent that thins mucus by breaking up the long strains of DNA commonly found in purulent sputum. Unfortunately not all CF patients can tolerate DNase. Although albuterol and tobramycin are used to treat cystic fibrosis, they are not mucokinetic agents, and thus not a substitute for DNase. Hypertonic saline (5-7%) is classified as a mucokinetic, and thins secretions by drawing extra water into the mucus layer lining the airways. The correct answer is: hypertonic saline

Over the course of 6 months, a patient whose asthma was not well-controlled has shown substantial improvement in both the frequency of symptoms and %predicted FEV1 with twice daily inhalation of a long-acting beta-agonist (LABA) and a medium dose inhaled corticosteroid. Which of the following would you recommend to the patient's doctor as possible changes to the current drug regimen? Select one: A. replace the LABA with a long-acting anticholinergic B. lower the dosage of the inhaled corticosteroid C. add a leukotriene modifier to the drug regimen D. replace the LABA with a short-acting beta agonist

Drug management of patient's with asthma is tailored to the severity of disease and its degree of control. If the patient's control of symptoms improves, drug treatment should STEP DOWN, eliminating drugs and/or reducing their dosages as the patient gets better. In this case the doctor could consider either discontinuing the long-acting beta-agonist or decreasing the inhaled corticosteroid dosage. A short-acting beta agonist (SABA) like albuterol should always be available for PRN use. However, regular daily use of SABAs can lead to tolerance and thus decrease their ability to effectively control symptoms. The correct answer is: lower the dosage of the inhaled corticosteroid

Which of the following drugs would you recommend for a patient with asystole? Select one: A. amiodarone B. epinephrine C. atropine D. magnesium

Epinephrine (1 mg IV push every 3-5 min) is the recommended drug for treating a patient with asystole or PEA. Atropine no longer is recommended for patients with asystole or PEA. Amiodarone (an antiarrhythmic) is used only for shockable rhythms and magnesium is used to treat torsades de pointes. The correct answer is: epinephrine

Which of the following measures is most consistent with an adult patient's readiness to wean from mechanical ventilation? Select one: A. spontaneous rate of 36/min B. vital capacity of 1.5 L C. spontaneous VT of 200 mL D. MIP/NIF of -15 cm H2O

Even though the patient's weight is not given, a vital capacity of 1.5 L is usually above the threshold required for weaning (> 10-15 mL/kg) for an average size patient (60-80 kg).The high respiratory rate and low tidal volume combine to yield a rapid shallow breathing index of 180 (36/0.2), well above the threshold of 100-105 that predicts weaning failure. Likewise, the low MIP value coincides with a low probability of weaning success. The correct answer is: vital capacity of 1.5 L

Which of the following treatments can increase a patient's susceptibility to infection? Select one: A. vasopressors B. supplemental O2 C. bronchodilators D. corticosteroids

Persons treated with corticosteroids, immunosuppressive agents or radiation therapy may be particularly susceptible to infection. Advanced age, chronic debilitating disease, shock, coma, traumatic injury, or surgical procedures also increase the susceptibility of patients to infection. The correct answer is: corticosteroids

A patient in ICU has been receiving volume control A/C ventilation via an oral endotracheal (ET) tube for 6 days. An attending 2nd year resident asks you whether you would recommend having the patient undergo tracheotomy. Which of the following should be considered in making your recommendation? Projected time artificial airway needed Patient's condition/ ability to undergo procedure Patient's tolerance of endotracheal tube A. Yes Yes No B. Yes No Yes C. No Yes Yes D. Yes Yes Yes

Factors to take into account when considering switching from an ET tube to a tracheostomy include (1) the projected time the patient will need an artificial airway; (2) the patient's tolerance of the ET tube; (3) the patient's overall condition (including nutritional, cardiovascular, and infection status); (4) the patient's ability to tolerate a surgical procedure; and (5) the relative risks of continued endotracheal intubation versus tracheostomy. The correct answer is: D

A patient with chronic bronchitis is receiving an aerosolized bronchodilator. The patient has a loose but nonproductive cough. You should recommend which of the following? Select one: A. Discontinue bronchodilator therapy and initiate chest physical therapy B. Increase the bronchodilator dose by 50% and start bland aerosol therapy C. Initiate positive expiratory pressure (PEP) therapy with bronchodilator therapy D. Instruct patient to perform slow, deep inspirations followed by a 2-sec breathhold

First, there is no information indicating a need to alter the bronchodilator dosage. Likewise, because there is no evidence of atelectasis, having the patient perform slow, deep inspirations with a breathhold (essentially the same as incentive spirometry) is not indicated. The presence of "a loose but nonproductive cough" indicates that this patient has a problem with secretion clearance. Although the bronchodilator may be helping open the patient's airways, additional therapy should be recommended to help mobilize and clear secretions. The most efficient way to accomplish this would be to combine the bronchodilator administration with a therapy that facilitates secretion clearance, such as positive expiratory pressure (PEP). The correct answer is: Initiate positive expiratory pressure (PEP) therapy with bronchodilator therapy

Which of the following secretion clearance methods would you recommend for a home care patient who lives alone? Select one: A. percussion and vibration B. postural drainage C. incentive spirometry D. PEP mask therapy

For a home care patient who need help with secretion clearance and lives alone directed coughing, forced exhalation, active cycle of breathing, and autogenic drainage methods can be used. Adjuncts like PEP mask therapy, the flutter valve and a percussion or oscillation vest could also be considered for the home-bound patient living alone. Caregiver assistance would be required for traditional postural drainage, percussion and vibration, and incentive spirometry and IPPB are lung expansion therapies, primarily indicated for atelectasis. The correct answer is: PEP mask therapy

A patient with neuromuscular disease has been on ventilatory support for 4 months via standard cuffed tracheotomy tube. At this point, she requires only nighttime ventilator support. Which of the following artificial airways should you recommend? Select one: A. tracheostomy button B. Bivona (foam cuffed) trach tube C. cuffed, fenestrated trach tube D. uncuffed, standard trach tube

For a patient with a tracheostomy on long-term mechanical ventilation who still requires intermittent support, a cuffed, fenestrated tracheostomy tube is the ideal airway. With the inner cannula in place and cuff inflated, a fenestrated tube performs like a standard tube and can be used for positive pressure ventilation. However, fenestrated trach tubes have one or more openings in their posterior curvature that permits air movement through the tube (from trachea to larynx) when the inner cannula is removed. This air flow allows talking and facilitates communication. In addition, by removing a fenestrated tube's inner cannula, plugging the outer cannula, and deflating its cuff, you can test the patient's ability to resume normal upper airway function. The correct answer is: cuffed, fenestrated trach tube

A patient receiving volume control A/C ventilation has the following ABGs on an FIO2 of 0.50: pH=7.58 PCO2= 24 torr HCO3 22 mEq/L PO2=70 torr The resident requests that you raise the patient's PCO2. Which of the following actions would be appropriate? Select one: A. decrease the set ventilator rate B. increase the inspiratory flow C. add mechanical deadspace D. increase the set tidal volume

For patients in acute respiratory alkalosis, the PaCO2 should be raised. When the patient is allowed to trigger the ventilator (assist/control), lowering the preset (machine) rate may not result in the desired rise in PaCO2. In these cases, one may have to add mechanical deadspace to the ventilator circuit. The correct answer is: add mechanical deadspace

A patient receiving airway pressure release ventilation (APRV) needs transport to/from the imaging center for a lengthy catheterization procedure. Which of the following would be the optimal plan to manage this patient throughout the transport and procedure? Select one: A. use a transport ventilator capable of full support and assist-control ventilation B. set up and apply the equivalent ventilator and mode in the receiving location C. bag the patient on 100% oxygen during transport and throughout the procedure D. recommend against transporting this patient due to the complexity of management

For patients requiring mechanical ventilation, the ideal plan is to have equipment available at the receiving location that is capable of delivering ventilatory support equivalent to that being delivered at the patient's origin. Occasionally patients may require modes of ventilation or ventilator settings not reproducible at the receiving location or during transportation. Under these circumstances, the origin location must trial alternate modes of mechanical ventilation before transport to ensure acceptability and patient stability with this therapy. The correct answer is: set up and apply the equivalent ventilator and mode in the receiving location

A 2 year-old child with a 24-hour history of an upper respiratory infection and low grade fever is admitted to the Emergency Department with audible inspiratory stridor. The stridor did not improve after cool bland aerosol therapy treatment. You should now recommend aerosol administration of: Select one: A. alfa dornase (Pulmozyme) B. pentamidine (NebuPent) C. racemic epinephrine D. albuterol (Proventil)

Given the child's age, history and clinical findings, the likely problem is viral croup or laryngotracheobronchitis. Stridor in croup is due to inflammation and edema of the subglottic larynx and trachea. With its strong alpha-1 receptor activity (causing vasoconstriction), racemic epinephrine is the drug of choice to treat airway edema. Pulmozyme is used to break thin thick, DNA-rich secretions, NebuPent is used to treat Pneumocystis pneumonia in HIV-infected patients and Proventil has no significant alpha activity. The correct answer is: racemic epinephrine

You are administering chest physical therapy in the Trendelenburg position to a 68-year-old male patient with chronic bronchitis. The patient starts to cough up a small amount of fresh blood. What action should you take? Select one: A. continue the therapy and provide supplemental oxygen B. continue the therapy but report the problem to the nurse C. stop the therapy, stabilize the patient up and contact a physician D. immediately call a code and begin resuscitation

Hemoptysis is always a serious problem. You should stop the treatment, stay with the patient until he is stable then and contact a physician. The correct answer is: stop the therapy, stabilize the patient up and contact a physician

During which of the following activation stages of a disaster should the respiratory therapist be available for immediate deployment? Select one: A. alert B. stand by C. call out D. stand down

Hospital personnel should be available for immediate deployment in the 'stand by' stage of disaster activation. During the alert stage, a disaster is possible and preparedness should be increased. The 'call-out' stage begins active deployment and the 'stand down' stage indicates that the disaster is contained. The correct answer is: stand by

After warming, positioning, suctioning and providing positive pressure ventilation for 30 seconds, a newborn infant's heart rate is 55/min. What should you do now? Select one: A. continue ventilation for another 30 seconds B. begin external cardiac chest compressions C. administer epinephrine and atropine D. provide supplemental O2, observe and monitor

If a newborn infant's heart rate is less than 60/min after warming, positioning, suctioning and providing positive pressure ventilation for 30 seconds, you should begin external cardiac chest compressions. The correct answer is: begin external cardiac chest compressions

An intubated patient on a ventilator is suspected of having suffered a pneumothorax, but is maintaining satisfactory blood pressure and adequate oxygenation (SpO2 = 89%). Which of the following initial actions would you recommend? Select one: A. obtaining a stat arterial blood gas B. immediately extubating the patient C. obtaining a stat chest X-ray D. beginning chest compressions

If a ventilator patient is suspected of having a pneumothorax but does not appear in immediate danger, a stat chest X-ray (or thoracic ultrasound) should be obtained to confirm the presence and assess the size of the lesion. At the same time the care team should be preparing for either needle decompression or chest tube insertion on the affected side. An ABG would not help confirm a pneumothorax and extubation might only further complicate the patient's management. The correct answer is: obtaining a stat chest X-ray

After several attempts at positioning the mask of a manual resuscitator on a bearded, edentulous male in the Emergency Department, you cannot provide adequate ventilation. Which of the following actions would you initially recommend to overcome this problem? Select one: A. surgical cricothyrotomy B. laryngeal mask airway insertion C. percutaneous tracheostomy D. noninvasive ventilation

If adequate ventilation cannot be provided by mask, the patient should have an advanced airway inserted. Advanced airways include endotracheal tubes and supraglottic airways, such as the laryngeal mask airway or King tube. Surgical intervention would be premature and needlessly time-consuming. The correct answer is: laryngeal mask airway insertion

Which of the following infants may require home apnea monitoring? Select one: A. infants with acyanotic congenital heart disease B. infants with bladder and bowel contrrol problems C. infants with one or more siblings who died of SIDS D. infants with BPD requiring mechanical ventilation

In addition to neonates at risk for recurrent apnea, bradycardia and hypoxemia after hospital discharge, home apnea monitoring may also be required for: infants receiving stimulants to treat apnea and bradycardia; infants with chronic lung disease requiring life support (e.g., oxygen, CPAP, mechanical ventilation); infants with gastroesophageal reflux if symptomatic with color and tone change; infants of substance abusing mothers if clinically symptomatic; infants with a tracheostomy or anatomic abnormalities that make them vulnerable to airway compromise; and infants with neurologic or metabolic disorders affecting respiratory control. Home apnea monitoring is NOT indicated to prevent sudden infant death syndrome (SIDS). The correct answer is: infants with BPD requiring mechanical ventilation

A physician orders intubation and volume control A/C ventilation for a 6 foot 3 inch tall 190 lb (86 kg) adult male patient with ARDS. Which of the following ventilator settings would you aim for to support this patient? Rate/min VT (mL) A 10 800 B 8 1200 C 20 900 D 15 500 Select one: A. A B. B C. C D. D

In adult patients with ARDS, the goal is to achieve a tidal volume of 6 mL/kg of predicted body weight, with a plateau pressure (Pplat) < 30 cm H2O. If Pplat > 30 cm H2O at 6 mL/kg, the VT can be reduced to as low as 4 mL/kg. The respiratory rate should be set to approximate the baseline minute ventilation, with the goal to achieve a pH between 7.20 and 7.45. For a six foot 3 inch tall patient with a predicted body weight of 85 kg, the goal would be a VT of about 500 mL with a rate between 10-20/min. In some cases rates as high as 35/min may be needed to maintain the pH above 7.30. The correct answer is: D

A 58-year-old patient in the open heart unit had cardiopulmonary bypass surgery with significant blood loss. On physical exam the patient presents with tachypnea and tachycardia and the SpO2 is 85% on 4 L/min nasal cannula. You should now recommend: Select one: A. non-rebreathing mask at 12 L/min B. CPAP with 5 cm H2O pressure C. nasal cannula at 6 L/min D. 60% air-entrainment mask

In combination, the tachypnea and tachycardia and the SpO2 all signal significant hypoxemia (an SpO2 of 85% equates to a PaO2 of 50 torr), compounded by diminished O2 carrying capacity due to the blood loss during surgery. To treat this hypoxemia, you should deliver the highest possible FIO2 to this patient. Among the available options, only the non-rebreathing mask is capable of consistently providing oxygen concentrations above 60%. The correct answer is: non-rebreathing mask at 12 L/min

While given an adrenergic aerosol bronchodilator treatment to an adult patient, you notice an increase in pulse rate from 85/min (pre-treatment) to 125/min. Which of the following would be the correct action in this case? Select one: A. terminate the treatment and record adverse response in chart B. use half the standard dosage listed in the package insert C. add more diluent (e.g., saline) to the nebulizer chamber D. temporarily stop the treatment and let the patient calm down

In general, an adrenergic bronchodilator should not be given if a patient already is experiencing tachycardia. Moreover, once a treatment is begun with an adrenergic agent, it should be terminated if the pulse rate increases more than 20 beats/min. The correct answer is: terminate the treatment and record adverse response in chart

A patient with history of CHF is admitted in the ICU. On physical assessment you note bilateral wheezing, severe pedal edema, hepatomegaly and jugular venous distension. The ICU resident wants your opinion on the management of his condition. Which of the following would you recommend? Select one: A. giving a bronchodilator treatment B. pushing an IV fluid bolus C. obtaining an arterial blood gas D. administering an inotropic agent

In patients with CHF, signs of fluid retention (wheezing, pedal edema, hepatomegaly and jugular venous distension) are common. The use of inotropic medications to increase heart contractility and diuretics to decrease the fluid overload are very important in the management of CHF, as is careful I&O monitoring. Because wheezing in CHF is usually due to peribronchial edema, bronchodilator therapy generally is not warranted. Rapid fluid administration would be contraindicated in this scenario and obtaining a blood gas does nothing to help manage the immediate problem at hand. The correct answer is: administering an inotropic agent

You would recommend oxygen therapy to treat: Select one: A. secondary polycythemia B. acute hypercapnia C. carbon monoxide poisoning D. absorption atelectasis

Indications for O2 therapy include (1) documented hypoxemia, acute care situations in which hypoxemia is common (e.g., shock, trauma, CO poisoning), acute myocardial infarction (to decrease myocardial workload) and short-term therapy for patients likely to develop hypoxemia (e.g., post-anesthesia recovery). Absorption atelectasis is a potential HAZARD of supplemental O2 therapy. The correct answer is: carbon monoxide poisoning

By doctor order, a patient with bacterial pneumonia has received postural drainage and percussion to all lung segments four times a day for one week. The pneumonia has cleared in all segments with the exception of the left lower lobe. At this time you should recommend: Select one: A. discontinuing the therapy B. decreasing the frequency of therapy C. continuing the therapy as ordered D. limiting the therapy to the involved segments

Postural drainage and percussion should always be limited only to affected areas of the lung. The correct answer is: limiting the therapy to the involved segments

After bronchodilator therapy, you record the following PFT data on a 67 year-old male COPD patient with chronic CO2 retention and respiratory acidosis: FEV1/FVC = 56%; FEV1 = 27% predicted. You would characterize the stage of the patient's COPD as: Select one: A. mild B. moderate C. severe D. very severe

Irreversible airflow obstruction is present when the FEV1/FVC ratio after bronchodilator treatment is less than 70% of predicted. The stage of COPD is then gauged by its impact on the predicted FEV1. If the FEV1 is < 30% predicted (or less than 50% predicted with chronic respiratory failure), the severity is classified as very severe. The correct answer is: very severe

After bronchodilator therapy, you record the following PFT data on a 67 year-old male COPD patient with chronic cough and sputum production: FEV1/FVC = 65%; FEV1 = 82% predicted. You would characterize the stage of the patient's COPD as: Select one: A. mild B. moderate C. severe D. very severe

Irreversible airflow obstruction is present when the FEV1/FVC ratio after bronchodilator treatment is less than 70% of predicted. The stage of COPD is then gauged by its impact on the predicted FEV1. If the FEV1 ≥ 80% the patient's predicted value and there is no significant SOB on exertion, the stage is classified as mild. The correct answer is: mild

Which of the following would you not recommend for a critically ill patient with signs and symptoms of fluid overload (overhydration)? Select one: A. initiate diuretic therapy B. restrict and closely monitor fluid intake C. administer steroids D. if renal failure, consider dialysis

Management of patients with fluid overload (overhydration) normally includes restriction of fluid intake, administration of diuretics and in critically ill patients insertion of a CVP catheter to closely monitor fluid balance. If heart failure is suspected, inotropic agents may be considered temporarily. If renal failure is a contributing factor, dialysis may be initiated. Corticosteroids tend to cause sodium and fluid retention. The correct answer is: administer steroids

A paralyzed patient being mechanically ventilated in the volume control mode has an abnormally low PaCO2 and high pH. Which of the following would be the best way to increase the patient's PaCO2 without changing the minute ventilation? Select one: A. decrease the tidal volume and increase the rate B. increase the tidal volume and decrease the rate C. add mechanical dead space to the ventilator circuit D. increase the inspiratory flow

Minute ventilation = alveolar ventilation + deadspace ventilation. By increasing the mechanical deadspace, you can decrease alveolar ventilation, raise the PaCO2 and lower the pH without changing minute ventilation. The correct answer is: add mechanical dead space to the ventilator circuit

Which of the following drugs would be most appropriate to recommend as a substitute for albuterol (Proventil) for a patient who has bronchospasm and whose cardiac rate increases substantially with each treatment? Select one: A. isoetharine (Bronkosol) B. terbutaline sulfate (Brethaire) C. racemic epinephrine D. ipratropium bromide (Atrovent)

Most sympathomimetic bronchodilators result in some beta-1 stimulation, causing an increase in heart rate. Though albuterol has fewer beta-1 effects than earlier adrenergic bronchodilators like isoetharine, it may result in a significant increased heart rate in some patients. In this instance, you might recommend an anticholinergic bronchodilator such as ipratropium bromide (Atrovent). The correct answer is: ipratropium bromide (Atrovent)

You are trying to apply mouth-to-mouth ventilation to an unconscious adult patient, but are unable to maintain a tight seal at the lips. At this point you should: Select one: A. place a handkerchief over the victim's mouth and continue B. use the jaw thrust maneuver instead of the head tilt/chin lift C. apply mouth-to-nose ventilation instead of mouth-to-mouth D. immediately apply a series of strong back blows to the victim

Mouth-to-nose ventilation is recommended if ventilation through the victim's mouth is impossible (eg, the mouth is seriously injured), the mouth cannot be opened, the victim is in water, or a mouth-to-mouth seal is difficult to achieve. The correct answer is: apply mouth-to-nose ventilation instead of mouth-to-mouth

Which of the following are prerequisites for successful application of noninvasive positive pressure ventilation (NPPV) in the management of chronic neuromuscular disease? Select one: A. intact upper airway function B. MIP > 25 cm H2O (neg) C. normal acid-base balance D. SpO2 > 90% on room air

NPPV is often used in managing the progressive respiratory acidosis that occurs in chronic neuromuscular diseases such as muscular dystrophies, postpolio syndrome and multiple sclerosis. Successful application requires a cooperative and motivated patient with intact upper airway function and minimal secretions. If patients can maintain normal acid-base balance and oxygenation on their own, NPPV probably is not needed. Similarly, a MIP > 25 cm H2O (neg) often is cited as one measure indicating muscle strength sufficient to maintain spontaneous ventilation. The correct answer is: intact upper airway function

In the absence of neck or facial injuries, the procedure of choice to establish a patent tracheal airway in an emergency is: Select one: A. orotracheal intubation B. surgical tracheotomy C. nasotracheal intubation D. cricothyrotomy

Orotracheal (oral) intubation is the procedure of choice in an emergency. This is because oral intubation, as compared to the nasal route or surgical tracheotomy, is the quickest and easiest means of establishing a tracheal airway The correct answer is: orotracheal intubation

For which of the following purposes would you recommend pressure support ventilation (PSV)? Select one: A. to help manage hypoxemic respiratory failure B. to control a patient's minute ventilation C. to increase the functional residual capacity D. to boost spontaneous volumes during SIMV

PSV is a spontaneous breath mode and therefore cannot be used to control a patient's minute ventilation. It can however be used to (1) help overcome the imposed work of breathing caused by artificial airways; (2) boost patients' spontaneous tidal volumes during SIMV; and (3) provide ventilatory assistance to patients during spontaneous breathing trials/weaning. By itself PSV does not increase FRC so it is not useful in managing hypoxemia due to shunting. The correct answer is: to boost spontaneous volumes during SIMV

A patient with a neuromuscular disorder who is breathing through an intact upper airway is receiving external (vest) oscillation to help mobilize secretions into the large airways, but is having difficulty clearing them. Which of the following techniques would you recommend to facilitate clearance of this patient secretions? Select one: A. manually assisted ("quad") cough B. postural drainage, percussion and vibration C. positive expiratory pressure (PEP) therapy D. acetylcysteine (Mucomyst) via aerosol

Patients with neuromuscular disorders can have trouble mobilizing and clearing secretions. To help mobilize secretion in patients with a neuromuscular disorder and an intact upper airway either postural drainage with percussion and vibration or external (vest) or internal (IPV) oscillation are indicated. Once mobilized, the secretions need to be cleared using one or more of the following methods: manually assisted cough, mechanical insufflation-exsufflation or nasotracheal suctioning. In these patients PEP therapy is not effective due to inadequate respiratory muscle strength (needed to generate the expiratory pressure). And acetylcysteine (Mucomyst) via aerosol may help thin/mobilize secretions, but does little to actually clear them from the large airways. The correct answer is: manually assisted ("quad") cough

A doctor orders bronchodilator therapy for a 170 lb adult male with a history of asthma who recently underwent gastric surgery. Your evaluation of the patient reveals moderate inspiratory and expiratory wheezing with an inspiratory capacity (IC) of 700 mL. Which of the following approaches would you recommend for administering the bronchodilator? Select one: A. breath-actuated metered dose inhaler B. CPAP or ExPAP with small volume nebulizer C. small volume nebulizer with mouthpiece D. multi-dose dry powder inhaler (DPI)

Postoperative patients--especially those having undergone upper abdominal procedures--are prone to developing atelectasis. Given that the patient's IC is well below that predicted for his size (about 3000-4000 mL), he is at particularly high risk for this complication. With the patient also in need of bronchodilator therapy (wheezing), the ideal approach therefore would be to combine an aerosolized beta-agonist like albuterol with a technique that can help maintain or increase the patient's FRC. For this reason you should recommend a positive airway pressure modality such as CPAP or EzPAP combined with bronchodilator drug delivery via small volume nebulizer The correct answer is: CPAP or ExPAP with small volume nebulizer

If the number of ventilators needed to support patients in respiratory failure after a chemical disaster is insufficient to meet the need, your initial response should be to: Select one: A. contact other local facilities and arrange for patient transfers B. call and order additional backup ventilators from the vendor C. enlist non-respiratory personnel to perform manual ventilation D. assign patients without needed equipment to triage priority Black

Preparedness planning for the respiratory care department includes estimating the number of patients who may need ventilatory support, determining number of staff members required to meet patient needs, and having in place a staff emergency call-back procedure. Should the available equipment and personnel not be able to meet patient needs, the first step would be to enlist non-respiratory personnel to perform manual ventilation using disposable BVMs. Once adequate immediate patient support is ensured, you can attempt to get additional backup equipment and/or make arrangements to transfer patients to other facilities. The correct answer is: enlist non-respiratory personnel to perform manual ventilation

Two days after thoracic surgery, a patient develops a low-grade fever, dyspnea, tachypnea and signs of infiltrates/consolidation in the lower lobes. The most likely cause of this problem is: Select one: A. pleural effusion B. pulmonary edema C. aspiration pneumonia D. pneumothorax

Pulmonary infiltrates frequently develop in patients after major thoracic or upper abdominal surgery, especially if performed under general anesthesia and/or if a pre-existing co-morbidity is present, e.g., COPD, obesity, etc. When accompanied by fever, dyspnea, and tachypnea, the most likely causes are a bacterial or aspiration pneumonia or atelectasis. The correct answer is: aspiration pneumonia

Two days after thoracic surgery, a patient develops a low-grade fever, dyspnea, tachypnea and signs of infiltrates/consolidation in the lower lobes. The most likely cause of this problem is: Select one: A. pleural effusion B. pulmonary edema C. bacterial pneumonia D. pneumothorax

Pulmonary infiltrates frequently develop in patients after major thoracic or upper abdominal surgery, especially if performed under general anesthesia and/or if a pre-existing co-morbidity is present, e.g., COPD, obesity, etc. When accompanied by fever, dyspnea, and tachypnea, the most likely causes are a bacterial or aspiration pneumonia or atelectasis. The correct answer is: bacterial pneumonia

A doctor wants an outpatient with idiopathic pulmonary hypertension to self-administer an inhaled pulmonary vasodilator. Which of the following drugs would you recommend? Select one: A. epoprostenol (Flolan) B. diltiazem (Cardizem) C. iloprost (Ventavis) D. bosentan (Tracleer)

Pulmonary vasodilators administered via the inhalation route currently include three prostacyclins: epoprostenol (Flolan), treprostinil (Tyvaso) and iloprost (Ventavis). Only treprostinil (Tyvaso) and iloprost (Ventavis) are approved for administration via the inhalation route, typically for use by outpatients. Inhaled epoprostenol (Flolan) is used off-label in the acute care setting as an alternative to inhaled nitric oxide therapy and typically administered via continuous nebulization using an IV drip into a jet nebulizer. Both diltiazem (Cardizem; a calcium channel blocker) and bosentan (Tracleer; an endothelin receptor antagonist) are used to treat pulmonary hypertension, but are administered orally. The correct answer is: iloprost (Ventavis)

Due to thick and tenacious secretions, a nurse is having difficulty suctioning a patient receiving ventilatory support via an 9 mm ET tube using a 14 French catheter. Humidification is being provided by an HME. Which of the following would you recommend to facilitate airway clearance in this patient? Assuring good systemic hydration Instilling 5 mL normal saline Switch to a heated humidifier Using a smaller suction catheter A. Yes Yes Yes Yes B. No Yes Yes Yes C. Yes No Yes No D. Yes Yes No Yes

Recommended interventions for managing thick tenacious secretions and preventing mucus plugs in ventilator-dependent patients are good systemic hydration and BTPS airway humidification via an active heated water humidifier. Instillation of saline may provoke a cough, but saline does not mix with mucous and may in fact be harmful, e.g., adversely affecting oxygenation and potentially dislodging bacterial colonies in the ET tube. For these reasons saline instillation currently is not recommend for routine use when suctioning. Instead, a trial of a mucolytic (e.g., acetylcysteine) might be considered, administered via aerosol (preceded by a bronchodilator) or by direct instillation into the ET tube. A smaller catheter will only increase the difficulty in clearing this patient's secretions. The correct answer is: C

Most restrictive lung diseases are characterized by: Select one: A. decreased flows and increased airway resistance B. decreased compliance and increased airway resistance C. decreased flows and decreased airway resistance D. decreased lung volumes and decreased compliance

Restrictive lung disease is characterized by decreased lung volumes. The primary factor in restrictive lung disease is a decrease in compliance (distensibility). This can be a result of changes in the lung tissues (parenchyma), the chest wall, or both. This decrease in lung compliance is usually the result of lung inflammation, fibrotic lung disease, neoplasms, or thoracic wall abnormalities such as kyphoscoliosis. The correct answer is: decreased lung volumes and decreased compliance

The PRIMARY indication for aerosolized ribavirin (Virazole) is in the treatment of: Select one: A. viral hepatitis B. herpes zoster C. respiratory syncytial virus D. influenza A and B

Ribavirin (Virazole) is an antiviral agent used to treat respiratory syncytial virus (RSV). Note that the American Academic of Pediatrics currently recommends against routine use of aerosolized ribavirin in infants or children with bronchiolitis. However, this drug may be helpful in managing patients with severe cases of bronchiolitis or those who are immunocompromised and/or have severe cardiopulmonary disease. Ribavirin also may be helpful in treating RSV infections in immuncompromized adults. The correct answer is: respiratory syncytial virus

A doctor orders a spontaneous breathing trial (SBT) for an 80 kg (176 lb) male patient intubated with a 6.5 mm endotracheal tube. Which of the following weaning strategies would you recommend? Select one: A. CPAP B. simple T-tube C. pressure support D. SIMV

SBTs can be conducted via simple T-tube or via 'on-ventilator' provision of CPAP and/or pressure support. Because this patient's endotracheal tube is smaller than normal for his size, the SBT should be implemented with pressure support, which can help overcome the extra work imposed by breathing through an artificial airway. Pressure support also can boost the VT and help keep respiratory rates in a manageable range. CPAP would be a consideration if either maintaining the patient's FRC or the occurrence of auto-PEEP was at issue. SIMV is not used to conduct spontaneous breathing trials. The correct answer is: pressure support

A patient with a history of COPD is brought to the emergency room receiving oxygen at 2 L/min via nasal cannula. You note that the patient is agitated and tachypneic. The patient has a palpable pulse. Which of the following should you recommend FIRST? Select one: A. sedating the patient B. intubation and mechanical ventilation C. increasing oxygen to 6 L/min D. obtaining an SpO2 measurement

Since agitation and tachypnea are potential signs of hypoxemia, and since hypoxemia is common in some COPD patients, you should immediately assess this patient's level of arterial oxygenation. Initially, this can be obtained via pulse oximetry (SpO2); however, to assure accuracy and establish baseline values, it will probably be necessary to get an ABG soon thereafter. The correct answer is: obtaining an SpO2 measurement

Despite an intensive regimen of positive airway pressure and airway clearance therapy, a postoperative patient continues to exhibit clinical manifestations of atelectasis due to large airway obstruction. The best treatment approach in this case would be: Select one: A. intubation and mechanical ventilation B. bedside therapeutic bronchoscopy C. transtracheal aspiration D. aerosol therapy with acetylcysteine

Should intensive positive airway pressure and airway clearance therapy fail in treating atelectasis due to large airway obstruction, therapeutic bronchoscopy is indicated. This procedure can usually be performed at the bedside with moderate sedation. The correct answer is: bedside therapeutic bronchoscopy

A victim of a motorcycle accident is being mechanically ventilated. The patient's arterial blood gas results and ventilator settings are below: Ventilator Settings: Blood Gases: Mode Vol Ctrl A/C pH = 7.36 VT 550 mL PaCO2 = 35 torr Rate 14/min HCO3 = 19 mEq/L FIO2 0.70 PaO2 = 40 torr PEEP 5 cm H2O SaO2 = 72% Based on this information, which of the following changes are appropriate? Select one: A. change to the SIMV mode B. increase PEEP to 10 cm H2O C. increase the FIO2 to 0.90 D. increase the tidal volume

Since the patient's acid-base status is acceptable (fully compensated metabolic acidosis), his ventilation should not be changed (changes may be needed as the metabolic acidosis is corrected). However, the patient has severe hypoxemia (PaO2 < 60 torr ; FIO2 > 0.60; P/F = 57), most likely due to severe physiologic shunting. Given that the patient already is receiving a high and potentially dangerous FIO2 (0.70), the best strategy at this time would be to increase the PEEP level. With the hypoxemia due to physiologic shunting, raising the FIO2 to 0.90 would have little benefit but could cause additional harm, i.e., O2 toxicity. The correct answer is: increase PEEP to 10 cm H2O

Which of the following is the most common cause of sinoatrial arrest? Select one: A. insertion of devices into the airway B. increased intracranial pressure C. arterial hypoxemia or hypercapnea D. overdose of anticholinergic agents

Sinus arrest can accompany the early stages of anesthesia or certain bodily manipulations that elicit a strong vagal reflex. These include insertion of devices into the airway (e.g., intubation, bronchoscopy, pharyngeal probing) and traction on thoracic organs during surgery. Certain medications also can provoke sinus bradyarrhythmias, such as beta-blockers and ACE inhibitors. Prolonged sinus arrest requires resuscitation, usually with administration of isoproterenol and/or atropine. A pacemaker sometimes is required despite medical therapy. The correct answer is: insertion of devices into the airway

Sodium bicarbonate administration during resuscitation would be most helpful in which of the following situations? Select one: A. in patients with hypoxic lactic acidosis B. in patients suffering drug overdose C. in patients with known hyperkalemia D. in patients suffering protracted arrest

Sodium bicarbonate (50 mEq over 5 minutes) is helpful during resuscitation only in patients with known hyperkalemia. Note that bicarbonate alone is less effective in reducing serum K+ than glucose plus insulin or nebulized albuterol (10 to 20 mg over 15 minutes). For this reason it is best used in conjunction with these medications. The correct answer is: in patients with known hyperkalemia

An alert 23 year old patient is admitted with a history of bronchiectasis and significant purulent sputum production (not associated with cystic fibrosis). Her chest X-ray shows infiltrates in most segments of the right lower lobe with auscultation revealing course rhonchi and wheezing in the same area. On 3 L/min nasal O2 she has a SpO2 of 94%. In addition to antibiotic therapy, what treatment would you recommend? Select one: A. albuterol via small volume nebulizer QID B. airway clearance therapy of patient's choosing C. incentive spirometry to IC 10 breaths hourly D. 35% oxygen by air-entrainment mask

Some form of airway clearance therapy is indicated in patients producing large volumes of sputum (> 20-30 mL/day). At a minimum, instruction in effective coughing technique (e.g., FET) should be provided to all such patients. Should secretion retention persist, other methods may be considered, including postural drainage, PEP therapy, intrapulmonary percussive ventilation (IPV), and high-frequency chest wall compression (HFCWC). Adjunct treatment with bronchodilators and/or mucokinetic agents such as bland aerosols or hypertonic saline also may be appropriate if bronchospasm or viscous secretions are impairing airway clearance. Ultimately, the technique(s) chosen should be based on the patient's preferences and ability to adhere to the method, as well as their effectiveness. In addition, the selected therapy ideally should be cost-effective and have minimal impact on any coexisting conditions and the patient's lifestyle. The correct answer is: airway clearance therapy of patient's choosing

Oxygen therapy may be justified for reimbursement in patients with PaO2s greater than 55 torr in the presence of which one of the following conditions? Select one: A. peripheral vascular disease B. poorly controlled asthma C. deep venous thrombosis D. congestive heart failure

Some oxygen therapy can be justified in patients with PaO2s > 55 torr or SaO2s ≥ 89% in association with cor pulmonale, congestive heart failure, or polycythemia (hematocrit ≥ 56%). The correct answer is: congestive heart failure

Which of the following are treatment options in patients suffering from supraventricular tachycardia (SVT)? Vagal stimulation Intravenous adenosine Synchronized cardioversion A. Yes Yes No B. Yes No Yes C. No Yes Yes D. Yes Yes Yes

Some patients with supraventricular tachycardia (SVT) respond to vagal stimulation via carotid artery massage. When drug intervention is necessary, agents that slow conduction through the AV node, like adenosine or a calcium channel blocker are indicated. If SVT causes cardiovascular instability, synchronized cardioversion is indicated. For patients with recurrent symptomatic SVT radiofrequency catheter ablation is considered the first-line 'curative' treatment. The correct answer is: D

Which of the following is the most common sign associated with partial airway obstruction due to vomit, blood or secretions in a patient's upper airway? Select one: A. snoring B. wheezing C. gurgling D. stridor

The most common sign associated with partial airway obstruction due to vomit, blood or secretions in a patient's upper airway is gurgling. Snoring is associated with loss of pharyngeal muscle tone, stridor indicates partial laryngeal obstruction and wheezing is associated with narrowing of the more distal airways due to bronchospasm or edema. The correct answer is: gurgling

A physician orders a spontaneous breathing trial (SBT) for a patient using 5/10 cm H2O CPAP + pressure support. After 45 minutes, her respiratory rate increases to 30/min and you observe some minor accessory muscle use. Which of the following would you recommend to alleviate this problem? Select one: A. raising the CPAP to 8 cm H2O B. putting her back on full ventilatory support C. increasing pressure support to 15 cm H2O D. recommending a sedative for anxiety

Tachypnea and accessory muscle use during an SBT indicate an increased work of breathing. As long as these changes are minor, you first action should be to increase the pressure support level. This will decrease the patient's spontaneous work of breathing, and help achieve a more efficient breathing pattern (higher tidal volume and lower rate). The correct answer is: increasing pressure support to 15 cm H2O

A 6-year-old child is intubated and being mechanically ventilated. Pertinent data are below: Ventilator Settings: Blood Gases: Mode SIMV Vol Ctrl pH = 7.46 VT 200 mL PaCO2 = 33 torr Rate 25/min HCO3 = 23 mEq/L FIO2 0.90 PaO2 = 54 torr PEEP 0 cm H2O SaO2 = 87% Based on these data, which of the following changes would you initially recommend? Select one: A. Initiating 5 cm H2O PEEP B. Decreasing the rate to 20/min C. Increasing the FIO2 to 1.0 D. Increasing the inspiratory time

The ABG indicates a mild respiratory alkalosis with severe hypoxemia (P/F = 60). Although decreasing the minute ventilation is indicated, the first action should be to correct the most serious problem, i.e., the refractory hypoxemia (low PaO2 with high FIO2). Because this level of hypoxemia is always associated with intrapulmonary shunting, it will not respond to additional oxygen. Instead, you need to apply PEEP level to open collapsed alveoli. The correct answer is: Initiating 5 cm H2O PEEP

A 90-kg male patient with a traumatic chest injury is receiving volume control ventilation (assist/control) with an FIO2 of 0.5, a set rate of 18, and a tidal volume of 600 mL. Due to patient triggering of machine breaths, the total respiratory rate is 28-30 breaths/min. Results of an arterial blood gas analysis are as follows: pH = 7.52 PaCO2 = 28 torr HCO3 = 22 mEq/L BE = -2 PaO2 = 81 torr SaO2 = 96% On the basis of these results, the most appropriate action is to: Select one: A. increase the FIO2 B. add mechanical deadspace C. increase the ventilator rate D. increase the tidal volume

The ABG indicates acute respiratory alkalosis, likely due to the extra machine breath the patient is triggering. Of the options listed only adding mechanical deadspace will raise the PCO2 (via rebreathing) and help normalize the pH. Other potential solutions (not listed) include switching to SIMV or sedating the patient. The correct answer is: add mechanical deadspace

A 4 day old, 35-week gestation male with symptoms of respiratory distress syndrome is on 10 cm H2O nasal CPAP with an FIO2 of 0.30. His vital signs, appearance, and fluid output have been stable for 12 hours. Umbilical artery blood gas results are as follows: pH = 7.43 PaCO2 = 42 mm Hg HCO3 = 27 mEq/L BE = +1 PaO2 = 129 mm Hg SaO2 = 99% Based on this information, you would recommend which of the following? Select one: A. maintain present therapy B. obtain a chest X-ray C. decrease CPAP to 5 cm H2O D. decrease the FIO2 to 0.25

The available data indicate that the baby is improving. The only problem is the hyperoxemia (PaO2 = 129 mm Hg), which is of concern in newborn infants. You could lower the PaO2 either by decreasing the FIO2 or decreasing the CPAP. Since the FIO2 is in the safe range, you should decrease the CPAP pressure to minimize complications. The correct answer is: decrease CPAP to 5 cm H2O

A 60 kg. (132 lb.) COPD patient is receiving SIMV with a VT of 500 mL at 9/min with an FIO2 of 0.35. Blood gases are as follows: pH = 7.36; PCO2 = 61 torr; HCO3 = 36 mEq/L; PaO2 = 64 torr. Which of the following changes would you recommend at this time? Select one: A. increase the IMV rate B. increase the FIO2 C. maintain settings D. increase the VT

The blood gas indicates a fully compensated respiratory acidosis. Since the pH is normal, you don't want to recommend changing any ventilatory parameters. In terms of oxygenation, a PaO2 of 64 torr (indicating an Hb saturation of at least 90%) is considered acceptable for a COPD patient. In fact, increasing the PaO2 in such patients above this level can cause hypoventilation. In summary, you should leave well enough alone and maintain these settings. The correct answer is: maintain settings

Data for a 80 kg (176 lb) patient receiving ventilatory support are: Ventilator Settings: Blood Gases: Mode Vol Ctrl A/C pH = 7.57 VT 600 mL PaCO2 = 25 torr Set Rate 12/min HCO3 = 22 mEq/L Actual Rate 18/min PaO2 = 106 torr FIO2 0.50 SaO2 = 98% Which of the following changes would be appropriate at this time? Select one: A. increasing the tidal volume B. decreasing the set (machine) rate C. adding 5 cm H2O PEEP D. adding mechanical deadspace

The blood gas indicates acute respiratory alkalosis with adequate oxygenation. For patients in acute respiratory alkalosis, the PaCO2 should be raised. In the volume control A/C mode, lowering the preset rate may not result in the desired rise in PaCO2, especially (as here) if the patient's rate exceeds the machine rate. In these cases, one may have to add mechanical deadspace to the ventilator circuit. The correct answer is: adding mechanical deadspace

A patient with a history of congestive heart failure is admitted to the ED with severe dyspnea, wheezing and watery, pink secretions. A first year resident orders supplemental O2 and albuterol (Proventil) via SVN. You should: Select one: A. suggest adding bronchial hygiene therapy to the bronchodilator regimen B. recommend diuresis and CPAP/BiPAP with O2 instead of albuterol C. implement the therapy as ordered and obtain an arterial blood gas D. recommend immediate intubation and mechanical ventilation

The clinical findings indicate that the patient likely is suffering from acute cardiogenic pulmonary edema, with wheezing (if present) typically due to peribronchial edema and not bronchospasm. Typical therapy includes diuresis, supplemental O2, CPAP/BiPAP (to decrease venous return/pulmonary capillary pressures) and administration of vasodilators (e.g., captopril, nitroprusside, nesiritide) and inotropes (e.g., dobutamine or milrinone). The correct answer is: recommend diuresis and CPAP/BiPAP with O2 instead of albuterol

A sleep study conducted on a 38 year old adult male patient reveals the following: an average of 12 apneic periods/hr, each lasting at least 20 seconds. During the apneic episodes, the patient does not make any effort to breathe. The patient also complains of chronic day-time sleepiness. Which of the following is the most likely cause of this patient's problem? Select one: A. obstructive sleep apnea B. central sleep apnea C. chronic hypothyroidism D. high intracranial pressure

The most likely cause of this patient's problem is central sleep apnea. Central sleep apnea is characterized by the occurrence of at least 5 or more apneic periods/hr (of 10 seconds or more), during which the patient usually do not make any effort to breathe. Central sleep apnea is most likely due to a defect in the CNS respiratory control mechanism. The correct answer is: central sleep apnea

What is the ideal route for the administration of most drugs used in emergency life support situations? Select one: A. a peripheral IV line B. an endotracheal tube C. intramuscular injection D. direct intracardiac injection

The ideal route for the administration of most drugs used in emergency life support is a peripheral IV line, followed by the intraosseous route. Lidocaine, epinephrine, atropine and naloxone (L-E-A-N) all can be administered via the endotracheal tube, but only if neither the IV or intraosseous routes are unavailable. The correct answer is: a peripheral IV line

A patient with chronic hypercapnia is receiving 28% O2 via an air entrainment mask. The nurse indicates to you that this patient tends to become cyanotic and confused during meals. What action would you recommend? Select one: A. recommend switching to an 35% air entrainment mask B. recommend switching to a cannula at 6 L/min during meals C. recommend that the patient be switched to tube feedings D. recommend switching to a cannula at 2 L/min during meals

The importance of continuous O2 therapy in patients with chronic hypercapnia cannot be overemphasized. Unlike low-flow devices such as cannulas, air entrainment masks must be removed for eating and drinking. Even a short halt in therapy in these patients can cause the PaO2 to fall rapidly. In order to ensure continuous oxygenation for patients when the mask must be removed, a cannula set at a flow producing an equivalent FIO2 should be used. The correct answer is: recommend switching to a cannula at 2 L/min during meals

During postural drainage of the left lower lobe, a patient complains of acute chest pain. Which of the following should you do? Select one: A. give the patient supplemental oxygen B. continue the treatment with the bed flat C. ask the nurse to administer pain medication D. discontinue the treatment and monitor the patient

The occurrence of a major unexpected and potentially harmful hazard effect such as acute chest pain is reason enough to discontinue any treatment. According to the AARC, when a patient exhibits pain during postural drainage, you should stop therapy, exercise care in moving the patient, and consult the ordering physician. The correct answer is: discontinue the treatment and monitor the patient

A patient who just suffered a severe closed head injury is placed on volume controlled A/C ventilation. Her ICP is 24 mm Hg. In addition to assuring good patient-ventilator synchrony, which of the following initial goals should you recommend for her early ventilatory support? Select one: A. SaO2 = 100%; PaCO2 25-30 mm Hg; PIP ≤ 30 cm H2O B. SaO2 = 90%; PaCO2 35-40 mm Hg; PIP ≤ 30 cm H2O C. SaO2 = 100%; PaCO2 25-30 mm Hg; PIP ≥ 30 cm H2O D. SaO2 = 100%; PaCO2 35-40 mm Hg; PIP ≤ 30 cm H2O

The overall goal of managing patients with closed head trauma is to prevent secondary injury by maintaining adequate cerebral perfusion pressure (CPP) and brain oxygenation. Because CPP = mean arterial pressure (MAP) - intracranial pressure (ICP), ventilatory care should aim to (1) maximize arterial oxygenation and (2) avoid actions that would either increase ICP or lower MAP. Goals therefore include maintaining an SaO2 of 100%; keeping the PaCO2 between 35-40 mm Hg (hypercapnia increases ICP); keeping the PIP ≤ 30 cm H2O (minimally affecting MAP); and assuring good patient-ventilator synchrony (helps prevent increases in intrathoracic pressure/ICP). Hyperventilation should only be considered if there is an acute deterioration in neurologic status that does not respond to standard brain trauma therapy, such as osmotic diuresis, CSF fluid drainage and sedation/neuromuscular blockade. The correct answer is: SaO2 = 100%; PaCO2 35-40 mm Hg; PIP ≤ 30 cm H2O

Data for a 55 kg (121 lb) patient receiving ventilatory support are: Ventilator Settings: Blood Gases: Mode Press Ctrl A/C pH = 7.54 Press Limit 25 cm H2O PaCO2 = 24 mm Hg Rate 18 HCO3 = 22 mEq/L FIO2 0.35 PaO2 = 75 mm Hg PEEP 0 cm H2O SaO2 = 93% Which of the following changes would be appropriate at this time? Select one: A. increasing the pressure limit B. increasing the rate C. decreasing the pressure limit D. decreasing the rate

The patient has acute respiratory alkalosis (excessive ventilation). Ventilation is adjusted by altering the minute volume (VE) to achieve a PaCO2 with a normal pH. To decrease the VE during pressure control ventilation, one can decrease the rate and/or decrease the pressure limit. Here the rate is slightly high but the pressure limit is within the normal range (20 - 30 cm H2O), so the best choice would be to decrease the rate. The correct answer is: decreasing the rate

A 45-year-old, 65 kg. (143 lb.) male is being mechanically ventilated after thoracic surgery. Pertinent data are below: Ventilator Settings: Blood Gases: Mode SIMV Vol Ctrl pH = 7.47 VT 500 mL PaCO2 = 34 torr Rate 12/min HCO3 = 24 mEq/L FIO2 0.40 PaO2 = 54 torr Flow 60 L/min SaO2 = 88% The most appropriate action is to: Select one: A. increase the tidal volume to 900 mL B. increase the frequency to 15/min C. increase the F102 to 0.50 D. increase the flow to 70 L/mi

The patient has adequate ventilation (mild respiratory alkalosis), so no increase in ventilation is warranted. However, the PaO2 of 54 mm Hg indicates a moderately hypoxemia. For this reason you should consider ways to increase the PaO2. Of the choices available only raising the FIO2 to 0.50 meets this requirement. The correct answer is: increase the F102 to 0.50

A patient weighing 70 kg (154 lb) is being weaned via pressure support ventilation. After 30 minutes arterial blood gas results and related data are as follows: Mode Press support pH = 7.22 FIO2 0.35 PaCO2 = 61 mm Hg Pressure limit 10 cm H2O PaO2 = 79 mm Hg Patient rate 36 HC03 = 25 mEq/L VT 330 mL BE = +2 mEq/L Which of the following changes would be appropriate in this situation? Select one: A. increase the FIO2 to 0.40 B. increase the rate C. add 5 cm H2O PEEP D. raise the pressure support level

The patient is hypercapnic (respiratory acidosis) and thus the minute ventilation should be increased. To increase the minute ventilation during pressure support ventilation you need to increase the pressure limit. This will tend to increase the spontaneous tidal volume and allow for more efficient ventilation at a lower rate. The correct answer is: raise the pressure support level

A 55-year-old 52 kg (114 lb) female with an acute closed head injury has just been placed on volume controlled A/C ventilation with a tidal volume of 600 mL and a set rate of 15 at an FIO2 of 0.35. Her ICP is 18 mm Hg. Blood gas data are as follows: pH = 7.53 PaCO2 = 26 torr PaO2 = 110 torr HCO3 = 21 mEq/L BE = -2 mEq/L Which of the following would you recommend? Select one: A. wean the patient B. decrease the tidal volume C. decrease the set rate D. maintain present settings

The patient's ICP is above normal (7-15 mm Hg), so therapy should be aimed at lowering it. Primary therapies used to lower ICP include osmotic diuresis, draining of CSF fluid and sedation.In the past, it was common to hyperventilate patients in order to promote cerebral vasoconstriction and decrease the ICP. We now know that although hyperventilation decreases ICP, the vasoconstriction can cause cerebral ischemia and worsen the initial injury. Instead current guidelines recommend aiming for a normal PaCO2 (35-40 torr), PIP ≤ 30 cm H2O, SpO2 > 95%, and good patient-ventilator synchrony. Given that the tidal volume is excessive for this patient (> 10 mL/kg), the best option for normalizing the PaCO2 would be to lower it. Care should be taken to avoid hypercapnia, which increases ICP. Likewise, you should try to avoid high levels of PEEP which can decrease mean arterial pressure, increase ICP, and thus decrease cerebral perfusion pressure. Hyperventilation should only be considered 1) to minimize ICP increases prior to procedures like suctioning, 2) in the presence of confirmed cerebral herniation, and 3) ss salvage therapy when high ICPs do not respond to standard treatment The correct answer is: decrease the tidal volum

Data for a 70 kg male patient receiving mechanical ventilation is as follows: Ventilator Settings: Blood Gases: Mode SIMV Vol Ctrl pH = 7.39 VT 550 mL PaCO2 = 36 torr Set Rate 6 HCO3 = 21 mEq/L FIO2 0.35 PaO2 = 65 torr PEEP 10 cm H2O SaO2 = 92% Off the ventilator, the patient has a spontaneous breathing rate of 23/min, with a spontaneous VT of 410 mL but his saturation quickly drops to 84%. Which of the following actions would be appropriate in this situation? Select one: A. change to 10 cm H2O CPAP B. increase the set respiratory rate C. increase the set tidal volume D. add mechanical dead space

The patient's blood gas indicates satisfactory ventilation (normal pH, PaCO2), and his breathing parameters off the ventilator indicate that he can maintain adequate spontaneous ventilation. However he still needs an elevated baseline positive pressure to maintain satisfactory oxygenation. In this case, you could switch the patient to CPAP at 35% to maintain oxygenation and allow the patient to breathe on his own. The correct answer is: change to 10 cm H2O CPAP

On reviewing the results of the attending physician's examination of a patient 24 hours after upper abdominal surgery, you note 'dull percussion note and bronchial breath sounds - left and right LL.' Which of the following should be considered for this patient's respiratory care plan? Lung expansion therapy Bronchodilator therapy Supplemental oxygen A. Yes Yes No B. No Yes Yes C. Yes No Yes D. Yes Yes Yes Select one: A. A B. B C. C D. D

The physical findings are consistent with either pulmonary infiltrates, atelectasis or consolidation of the lower lobes. Options to consider in formulating a respiratory care plan for this patient should include lung expansion therapy and supplemental O2 as needed to maintain adequate O2 saturation. If excess secretions are contributing to the problem, airway clearance therapy also may be indicated. Because there are no signs of bronchospasm, bronchodilator therapy is not indicated. The correct answer is: C

The most reliable indicator of effective external chest compressions during resuscitation is: Select one: A. a palpable pulse with each compression B. contraction of previously dilated pupils C. a large positive "spike" on the EKG monitor D. greater ease of sternal compressions

The presence of a pulse is the most reliable indicator of effective chest compressions. In adults and children you should assess the carotid or femoral pulse; in infants and neonates, you can more easily palpate the brachial pulse. The correct answer is: a palpable pulse with each compression

The primary cause of bronchospasm, mucosal edema, and retained secretions is: Select one: A. inflammation B. CNS stimulation C. arterial hypoxemia D. neoplasia

The primary cause in the development of bronchospasm, mucosal edema, and retained secretions is the inflammatory response due to immune reactions or infection. The correct answer is: inflammation

Which one of the following is a goal of airway clearance therapy? Select one: A. to help mobilize retained secretions B. to reverse the underlying disease process C. to reduce the physiologic deadspace D. to increase the functional residual capacity

The primary goal of airway clearance therapy is to help mobilize and remove retained secretion, with the ultimate aim to improve gas exchange and reduce the work of breathing. The correct answer is: to help mobilize retained secretions

Home apnea monitoring of infants is medically indicated for which one of the following groups? Select one: A. infants discharged prior to 40 weeks postconceptual age B. infants diagnosed with cyanotic congenital heart disease C. infants who required surfactant therapy during hospitalization D. infants with BPD requiring supplemental oxygen and CPAP

The primary indication for using an apnea monitor is to identify and warn of life-threatening events in neonates at risk of recurrent apnea, bradycardia and hypoxemia after hospital discharge. Other conditions that may require home apnea monitoring include: 1) infants receiving aminophylline or caffeine therapy for a history of apnea and bradycardia; 2) infants with chronic lung disease (bronchopulmonary dysplasia), especially those requiring supplemental oxygen, continuous positive airway pressure, or mechanical ventilation; 3) infants with gastroesophageal reflux (GER) if symptomatic with color and tone change; 4) infants of substance abusing mother if clinically symptomatic; 5) infants with a tracheostomy or anatomic abnormalities that make them vulnerable to airway compromise; and 6) infants with neurologic or metabolic disorders affecting respiratory control. The correct answer is: infants with BPD requiring supplemental oxygen and CPAP

The proper rate of external chest compressions for infants during resuscitation is: Select one: A. 60-80/min B. 80-100/min C. ≥ 100/min D. 70-80/min

The proper rate of cardiac compressions for all age groups is at least 100/min. Compression depth for adults should be at least 2 inches (5 cm), for children at least 1/3 the AP chest diameter (about 2 inches), and for infants also at least 1/3 the AP chest diameter (about 1-1/2 inches). The correct answer is: ≥ 100/min

Which of the following airway clearance techniques would you recommend for a 15 month old infant with cystic fibrosis? Select one: A. postural drainage, percussion, and vibration B. positive expiratory pressure (PEP) therapy C. mechanical insufflation-exsufflation D. intrapulmonary percussive ventilation

The recommended airway clearance technique for a 15 month old infant with cystic fibrosis is postural drainage, percussion, and vibration. All other options require either cooperation or understanding that cannot be expected from a 15-month old or equipment designed for adults only. The correct answer is: postural drainage, percussion, and vibration

You are asked to assist in transporting an intubated critically ill patient receiving 40% oxygen via T-tube to radiology, and to stay with the patient and bring her back to ICU. Total transport time (to radiology and back) is about 20 minutes and the procedure is expected to take an additional 20 minutes. What is the minimum O2 supply (in minutes) that you should plan on having available? Select one: A. 40 minutes B. 50 minutes C. 60 minutes D. 70 minutes

The recommended minimum oxygen supply for transporting a patient is that projected as needed for the transport itself and procedures plus a 30-minute reserve. In this case 20 min transport + 20 min procedure + 30 min reserve = 70 minutes total. The correct answer is: 70 minutes

A newborn infant receiving a moderate concentration of O2 in an isolette must be removed in order to insert an umbilical artery catheter. Which of the following devices would you recommend to provide supplemental O2 to the infant during this procedure? Select one: A. catheter B. simple mask C. cannula D. oxyhood

The simple mask is the device of choice for delivering short-term O2 therapy to infants during transport or when they must be removed from an O2 enclosure for special procedures. However, masks are usually not well tolerated by infants, and can easily result in pressure necrosis of their delicate skin. The correct answer is: simple mask

Theophylline therapy would tend to increase the risk of exacerbating which of the following conditions? Select one: A. sinus tachycardia B. chronic asthma C. emphysema D. chronic bronchitis

Theophylline should be used with extreme caution in patients with cardiac tachyarrhythmias, active peptic ulcer disease, or seizure disorders due to the increased risk of exacerbating these conditions. In patients with COPD and chronic asthma, low-dose theophylline therapy has an anti-inflammatory effect and can help prevent exacerbations by decreasing dyspnea, air trapping, and the work of breathing. Theophylline also can improve contractility of the diaphragm. The correct answer is: sinus tachycardia

You would recommend placement of an artificial tracheal airway to: Select one: A. enable negative pressure ventilation B. decrease auto-PEEP C. protect against pulmonary aspiration D. increase anatomic deadspace

There are four basic indications for artificial tracheal airways: 1) to relieve airway obstruction; 2) to facilitate secretion removal; 3) to protect the lower airways from aspiration; and 4) to facilitate application of positive pressure ventilation. Negative pressure ventilation normally requires an intact and normally functioning upper airway and tracheal airways actually DECREASE anatomic deadspace. The correct answer is: protect against pulmonary aspiration

On entering a room to check on a patient, you notice the patient's roommate laughing at a television program while eating dinner. Suddenly she cannot speak and makes urgent motions with her hands to her mouth. You should: Select one: A. lean her forward and strike her several times between the shoulder blades B. perform the Heimlich maneuver (abdominal thrusts) on her C. check for a carotid pulse, call a code and begin resuscitation D. lay her back in bed and strike her several times over the sternum

This patient apparently has an acute airway obstruction due to the aspiration of food into her larynx or trachea. In these cases, the basic protocol is to perform the Heimlich maneuver immediately. The correct answer is: perform the Heimlich maneuver (abdominal thrusts) on her

A patient receiving bi-level positive airway pressure for acute respiratory failure has a PaO2 of 48 torr on 65% O2 and a PaCO2 of 38 torr with IPAP = 20 cm H2O and EPAP = 5 cm H2O. To raise this patient's PaO2, you should recommend: Select one: A. increasing the oxygen concentration to 80% B. increasing IPAP to 25, keeping EPAP at 5 cm H2O C. increasing EPAP to 10 and IPAP to 25 cm H2O D. increasing EPAP to 10, keeping IPAP at 20 cm H2O

This patient's hypoxemia is due to shunting (PaO2 ≤ 50 torr, FIO2 ≥ 0.50; P/F = 74). If shunting is present when administering NPPV for acute respiratory failure, you should increase the EPAP level. To maintain the same level of ventilation (PaCO2 = 38 torr), you would also want to keep DP (IPAP - EPAP) constant. In this case that would mean increasing the IPAP to 25 cm H2O. The correct answer is: increasing EPAP to 10 and IPAP to 25 cm H2O

Which of the following equipment would you use to suction the nose and oropharynx of a normal term infant delivered vaginally? Select one: A. meconium aspirator B. bulb syringe C. 8 Fr suction catheter D. ET tube

To suction the nose and oropharynx of a normal term infant delivered vaginally, you would use a bulb syringe. The correct answer is: bulb syringe

Which of the following drugs can be administered via an endotracheal tube during emergency life support? Lidocaine Amiodarone Epinephrine Atropine A. Yes Yes Yes No B. Yes No Yes Yes C. Yes Yes Yes No D. No Yes No Yes Select one: A. A B. B C. C D. D

Unless a central vein is already open, the ideal drugs route during emergency life support is a peripheral IV line. Lidocaine, epinephrine, atropine and naloxone (L-E-A-N) can be given through an endotracheal tube. An alternative mnemonic (N-A-V-E-L) includes vasopressin (adults only). Typically the dose given by the endotracheal route is 2 to 2-1/2 times the recommended IV dose, diluted in 5 to 10 mL of sterile water or normal saline and injected directly into the ET tube. The correct answer is: B

A doctor is considering weaning a patient from invasive mechanical ventilation. Which of the following approaches would you recommend to wean this patient? Select one: A. airway pressure release ventilation B. pressure control with PEEP C. pressure support with CPAP D. decreasing rate SIMV

Weaning is most quickly accomplished via daily application of spontaneous breathing trials (SBTs). SBTs typically are implemented via simple T-piece, CPAP or pressure support with CPAP. Incremental decreases in SIMV rate would be a consideration only if the patient were deemed ventilator-dependent; otherwise it only tends to lengthen weaning time. Pressure control (pressure-limited, time-cycled ventilation) is not used for weaning. The correct answer is: pressure support with CPAP

A patient on mechanical ventilation is improving, and the physician wants to begin weaning her. Originally, she was on pressure controlled A/C with a rate of 12/min and a tidal volume of 600 mL. For weaning the doctor orders a switch to pressure support ventilation at 10 cm H2O. Initially, the patient seems comfortable on pressure support, but after 45 minutes she is moderately tachypneic and is starting to use her accessory muscles. Which of the following would you recommend to alleviate this problem? Select one: A. giving her 10 mg of Valium for anxiety B. putting her back on pressure control ventilation C. increasing the pressure support level to 15 cm H2O D. adding 5 cm H2O of PEEP

When a patient being weaned exhibits tachypnea and accessory muscle use, it is likely that the work of spontaneous breathing is too high. In these cases, you first action should be to increase the pressure support level. This will allow for a decrease in spontaneous work of breathing, and a more efficient breathing pattern (higher tidal volume and lower rate). The correct answer is: increasing the pressure support level to 15 cm H2O

A home care patient with COPD who lives alone produces in excess of 30 mL of sputum daily and is having difficulty clearing his airway. Which of the following therapies would you recommend for this patient? Select one: A. incentive spirometry q2h followed by a bronchodilator B. postural drainage, percussion and vibration QID C. oscillation vest therapy followed by huff coughing PRN D. placement of a permanent tracheostomy button

When a patient has excessive secretions (> 30 mL/day) and difficulty removing them, airway clearance therapy is indicated. Postural drainage, percussion and vibration is the traditional approach, but is not possible here because the patient lives alone. Instead, a high frequency oscillation vest combined with the FET could be considered. Alternatively, teaching the patient a regimen of PEP therapy combined with autogenic drainage or the forced expiratory technique (FET)/huff coughing would be a good choice. The correct answer is: oscillation vest therapy followed by huff coughing PRN

You are administering an aerosolized albuterol (Proventil) treatment to a patient when you note he complains of dizziness, heart palpitations and feeling flushed. You next action should be to do which of the following? Select one: A. continue the therapy but recommend reducing its frequency B. stop the therapy, go to the nurse's station and review the chart C. continue the therapy, but add saline to further dilute the albuterol D. stop the therapy, monitor the patient and inform the patient's doctor

When patients experience an apparent adverse response to therapy, the best course of action normally is to stop the treatment, monitor their condition and advise the nurse and doctor. The correct answer is: stop the therapy, monitor the patient and inform the patient's doctor

While administering an IPPB treatment to a post-op abdominal surgery patient, you note that her neck veins become distended and her heart rate increases and becomes a bit irregular. What action would you take at this time? Select one: A. decrease the pressure setting B. stop the treatment/monitor the patient C. increase the delivered FIO2 D. increase the sensitivity setting

Whenever a patient develops an unexpected response or exhibits an unwanted side effect to therapy you are administering, you should immediately Stop the treatment, Stay with the patient, and monitor the patient until he or she is again Stable (Triple S rule - Stop, Stay, Stabilize) The correct answer is: stop the treatment/monitor the patient

On entering a patient's room to administer a treatment, you find the patient lying on the floor. After verifying that the patient is unresponsive you should: Select one: A. run to the nursing station for help B. call for help and a defibrillator C. go to the nearest telephone and call the operator D. wait until someone comes into the room

Whenever you find a patient slumped over in bed and unresponsive, you should immediately 1) activate the emergency system (call a code), 2) call for a defibrillator and 3) assess the patient's cardiopulmonary status. If the patient is pulseless, you should immediately begin chest compressions (C-A-B). The correct answer is: call for help and a defibrillator

You would prematurely end an IPPB treatment if the patient: Select one: A. coughs up purulent sputum B. leaks air through the nose/mouth C. spontaneously triggers each breath D. develops gastric distention

Whenever you observe a finding consistent with a potential hazard of a treatment you are administering, you should stop the therapy and stabilize the patient. Potential hazards of IPPB therapy that warrant termination of the treatment include increased airway resistance/work of breathing, barotrauma, hemoptysis, gastric distention, impaired venous return, and air trapping. Coughing up purulent sputum may be a positive outcome if the goal of therapy is to facilitate airway clearance. Leaks are a common problem that can be rectified with proper training or accessory devices like noseclips. And of course by definition patient should spontaneously triggers each IPPB breath. The correct answer is: develops gastric distention

You would prematurely end an IPPB treatment if the patient: Select one: A. leaks air out the nose B. coughs up blood C. triggers each breath D. coughs up sputum

Whenever you observe a finding consistent with a potential hazard of a treatment you are administering, you should stop the therapy and stabilize the patient. Potential hazards of IPPB therapy that warrant termination of the treatment include increased airway resistance/work of breathing, barotrauma, hemoptysis, gastric distention, impaired venous return, and air trapping. Coughing up sputum may be a positive outcome if the goal of therapy is to facilitate airway clearance. Leaks are a common problem that can be rectified with proper training or accessory devices like noseclips. And of course by definition patient should spontaneously triggers each IPPB breath. The correct answer is: coughs up blood

Following successful resuscitation after a severe myocardial infarction, a 66-year-old 70 kg male is placed on volume control A/C ventilation with the following settings: FIO2 0.65 Mandatory Rate 10/min Tidal Volume 600 mL PEEP 10 cm H2O The patient's blood pressure is 90/55 mm Hg and his SpO2 is 80%. Which of the following should you recommend first? Select one: A. Increase the mandatory rate to 15/min B. Increase the FIO2 to 1.0 C. Increase the PEEP to 15 cm H2O D. Decrease the tidal volume to 400 mL

With an SpO2 of 80% corresponding to a PaO2 of about 50 torr (and thus a P/F ration < 100), the patient has severe hypoxemia. Given the patient's compromised cardiac status (low blood pressure post-CPR), increasing PEEP may further hinder cardiac function and tissue perfusion. Although 100% O2 over the long term can be hazardous, a temporary increase in FIO2 to 1.0 should help improve oxygenation while that patient's other problems are addressed. The correct answer is: Increase the FIO2 to 1.0

Dozens of workers exposed to an unknown substance at large industrial site have become ill and are being brought to your hospital. You are asked to assist in decontaminating the victims. What personal protective equipment (PPE) would you wear? Select one: a. standard precaution PPE with splash protection for eyes b. encapsulated suit with a self-contained breathing apparatus c. airborne precaution PPE, including a fitted N-95 respirator d. chemical-resistant suit and gloves with an air-purifying respirator

Your answer is correct. It is generally agreed that personnel who carry out hospital-based decontamination should wear Level C PPE. Level C PPE includes a nonencapsulated, chemical-resistant suit, gloves, boots, and a full-face air-purifying respirator. The correct answer is: chemical-resistant suit and gloves with an air-purifying respirator


Set pelajaran terkait

Article 110- Requirements for Electrical Installations

View Set

International Economics Chapters 7,8,9

View Set

Wordly Wise 3000, book 7, lesson 9

View Set

AP Lab 3 Spongy and Compact bone

View Set

6. The Civil Rights Movement of the 1960s

View Set

Chapter 11 (Meiosis: The Cellular Basis of Sexual Reproduction)

View Set

Business Law 2, Test 1 - Chapter 21

View Set